博弈論

尋找純策略子博弈完美納什均衡

  • December 14, 2020

我有興趣找到下面遊戲的純策略子博弈完美納什均衡。令我困惑的是,當玩家 A 在減少或不減少他的最終收益之間做出選擇之後,遊戲不再是連續的,而是同時進行的。如何找到子博弈完美的納什均衡?

我想首先在兩個同時移動的子博弈中找到納什均衡,我總共得到了 4 個均衡(寫成結果):

$ (2,4) $ 和 $ (4,2) $ 對於左子遊戲,和

$ (1,4) $ 和 $ (3,2) $ 為正確的子遊戲。

由此,我根據可能的收益推斷出玩家 A 在第一階段會選擇什麼(不減少或減少),因此我得到了以下兩個子博弈完美納什均衡:

$ {Don’t\space reduce, a_2, b_2} $ , 和

$ {Reduce,a_2,b_2} $ .

我不確定這是否正確。在確定子博弈中的納什均衡後,我很困惑試圖確定玩家 A 在第一階段會做什麼。我這樣做對嗎?有人可以解釋如何解決這個問題嗎?

在此處輸入圖像描述

因為您正在尋找子博弈完美均衡,所以反向求解該博弈是正確的方法。有 2 個正確的子博弈,您在那裡確定了正確的納什均衡。但是,請注意您指定了收益,而不是策略。正確的表述是 $ (a_1,b_1) $ 代替 $ (2,4) $ .

接下來,您將子博弈完美收益替換為子博弈,並在開始時檢查 A 的最優選擇。重要的是,均衡始終是完整的策略配置文件。也就是說,它需要為每個資訊集指定一個動作,而不管它是否在平衡路徑上達到。A有三個動作,一個在開始,一個在左邊,一個在右邊子博弈。B 有兩個動作,每個子博弈一個。因此,策略配置文件將採用這種形式

$$ (first,left,right),(left,right) $$. 您必須檢查所有組合。如果你考慮左邊的收益 (2,4) 和右邊的 (1,4),你有一個 SPNE

$$ (don’t reduce,a1,a1),(b1,b1) $$. 接下來,考慮 (2,4) 和 (3,2),您有一個 SPNE$$ (reduce,a1,a2),(b1,b2) $$. 您還有兩個 SPNE:$$ (don’t,a2,a1),(b2,b1) $$和$$ (don’t,a2,a2),(b2,b2) $$. 也就是說,有四個SPNE。 如果您不指定偏離路徑會發生什麼,您將無法評估當玩家 A 在第一階段偏離時會發生什麼,因此無法確定是否不存在有利可圖的偏離。

引用自:https://economics.stackexchange.com/questions/41475